Discussion

Each of the following could be a complete and accurate list of the researchers who learn both Swahili and Yoruba EXCEPT:
(A)the historian
(B)...
(C)...
(D)...
(E)...
(F)...
*This question is included in Assignment Lesson: Set 1 (of 2) - Intro, question #22

The solution is

Posted: 01/21/2013 13:44
I don't understand why c is the answer. I can get a solution where it COULD be the answer, and I don't know how it is not complete
Posted: 01/21/2013 14:05
I can't seem to get a situation where the palaeontologist is the only person to study S and Y.

Since Y has to be studied by 3 researchers, doesn't that mean the 3 researchers are H, L and P (because if G studied Y, than so would L or P, which can't be possible bc of rule 4.). So if HLP are studying Y, than that would mean that G would have to study S - which can't happen by of rule 4. I swear this has to be the answer that cannot be true, but the results key says that C is the correct answer.
Posted: 01/24/2013 19:03
P Adams, you are right, sir. This is a tricky one to set up. The key from LSAC is C, and we agree with you that it is not the right answer. In fact, answer C is essentially the same as answer D.

The right key should be B. Thanks to user Andrew Mandelbaum who brought it to our attention.
Posted: 01/24/2013 19:32
Key words: "complete and accurate list", which means no other researchers can have S & Y checked, other than the researcher(s) mentioned in the answer choice. Graph explanation to follow.
Posted: 01/24/2013 19:56
This is the chart to set up the problem for answer choice B. As we fill out the matrix, following the rules, we will eventually violate rule 2, that there must be 2 researchers learning Swahili.

To fill out the matrix, check PS and PY (P is studying S and Y).
Then cross GR and HR, since there can only be 1 check in R, and if G is checked, so must be H.
Next, cross GS and GY, since L or P --> ~G.
Next, check GT, because G has to learn at least 1 language. By extension, check HT, because G -->H.
Next, check HY and LY, since there has to be 3 x Y. Also, cross LT and PT, because there can only be 2 x T.

Now, the moment of truth. Cross HS and LS, because the answer must be a "complete and accurate" list, which means that if P is the only researcher that is studying S and Y, then H and L cannot be studying S (previously we established that H and L are studying Y).

However, this will violate the condition that there must be 2xS. Hence, B cannot be the correct answer.
Posted: 01/24/2013 22:20
Thanks. I can now sleep easily in knowing that B was the correct answer :).
Image Not Available
Contributor
Posted: 01/24/2013 22:43
P Adams, u r welcome. When r u planning to take LSAT and what are your preparation methods other than this app?

You need to be signed in to perform that action.

Sign In